SMO Primes!

Let p p , q q and r r be prime numbers such that

p q r = 19 ( p + q + r ) . pqr=19(p+q+r).

What is the value of p 2 + q 2 + r 2 ? p^2+q^2+r^2?


This problem appeared in the SMO - 2014.
This problem is a part of the set "Olympiads and Contests Around the World - 2." You can see the rest of the problems here .


The answer is 491.

This section requires Javascript.
You are seeing this because something didn't load right. We suggest you, (a) try refreshing the page, (b) enabling javascript if it is disabled on your browser and, finally, (c) loading the non-javascript version of this page . We're sorry about the hassle.

9 solutions

Paola Ramírez
Jan 25, 2015

without loss of generality p = 19 p=19 becasuse 19 p q r 19|pqr also can be r r o q q .

q r = 19 + q + r qr=19+q+r

( q 1 ) ( r 1 ) = 20 (q-1)(r-1)=20

q 1 = 1 , 2 , 4 , 5 , 10 , 20 ) q q-1=1,2,4,5,10,20) \Rightarrow q can be 2 , 3 , 5 , 11 2,3,5,11

If q = 2 r = 21 q=2 \Rightarrow r=21 so r r is not a prime.

If q = 3 r = 11 q=3 \Rightarrow r=11 so r r is a prime. \star

If q = 5 r = 6 q=5 \Rightarrow r=6 so r r is not a prime.

If q = 11 r = 3 q=11 \Rightarrow r=3 so r r is a prime. \star

So p = 19 , q = 3 , r = 11 p 2 + q 2 + r 2 = 491 p=19, q=3, r=11 \therefore \boxed{p^2+q^2+r^2=491}

Allow me to expand your brilliant solution.

We have: p q r = 19 ( p + q + r ) pqr=19(p+q+r)

Since we have a product of three primes on the left hand side, let's look at the factors of the right hand side. Note that 19 19 is the first factor. Therefore p q r pqr is divisible by 19 19 .

Note that 19 19 is a prime. Therefore, according to the Fundamental Theorem of Arithmetic , one of p p , q q , r r must be 19 19 . Without loss of generality, let p p be 19 19 .

The equation now becomes: 19 q r = 19 ( 19 + q + r ) 19qr=19(19+q+r)

Simplify it to obtain: q r = q + r + 19 qr=q+r+19

Arrange so that only constant is on the right hand side: q r q r = 19 qr-q-r=19

The left hand side cannot be completely factorized now: q ( r 1 ) r = 19 q(r-1)-r=19

Add one to both sides to make ( r 1 ) (r-1) appear so as to obtain common factor: q ( r 1 ) ( r 1 ) = 19 + 1 q(r-1)-(r-1)=19+1

Factorize: ( q 1 ) ( r 1 ) = 20 (q-1)(r-1)=20

From this we can see that ( q 1 ) (q-1) and ( r 1 ) (r-1) are factors of 20 20 .

Without loss of generality, we can let ( q 1 ) (q-1) be the smaller factor.

Factors of 20 20 : 1 1 , 2 2 , 4 4 , 5 5 , 10 10 , 20 20

  • q 1 = 1 r 1 = 20 q = 2 , r = 21 q-1=1\implies r-1=20\implies q=2,r=21
  • q 1 = 2 r 1 = 10 q = 3 , r = 11 q-1=2\implies r-1=10\implies q=3,r=11
  • q 1 = 4 r 1 = 5 q = 5 , r = 6 q-1=4\implies r-1=5\implies q=5, r=6

Only q = 3 , r = 11 q=3,r=11 give us both primes.

Therefore, p = 19 p=19 , q = 3 q=3 , r = 11 r=11 , and p 2 + q 2 + r 2 = 491 p^2+q^2+r^2=491 .

===========================================

Notes:

Other solutions are:

  • p = 19 p=19 , q = 11 q=11 , r = 3 r=3
  • p = 11 p=11 , q = 19 q=19 , r = 3 r=3
  • p = 11 p=11 , q = 3 q=3 , r = 19 r=19
  • p = 3 p=3 , q = 11 q=11 , r = 19 r=19
  • p = 3 p=3 , q = 19 q=19 , r = 11 r=11

Kenny Lau - 5 years, 10 months ago

Log in to reply

Great solution.!!

Chaitnya Shrivastava - 5 years, 8 months ago

I did the same way!

Sarith Imaduwage - 5 years, 4 months ago

Even i had the same soln.....

Aditya Kumar - 5 years, 1 month ago

Why p = 19? Why not 38, 57, ..., (19k) ?

Muhammad Irfan Wicaksono - 4 years, 5 months ago

Log in to reply

p is a prime

Henry U - 2 years, 8 months ago

Exactly how I solved. Well done

Nitin Kumar - 1 year, 3 months ago
Jubayer Nirjhor
Jun 6, 2014

First note that the expression is completely symmetric. Since ( p , q , r ) P 3 (p,q,r)\in\mathbb P^3 , we let p = 19 p=19 . Then we get q r = 19 + q + r qr=19+q+r . Rearranging and factorizing gives us ( q 1 ) ( r 1 ) = 20 = 2 × 10 (q-1)(r-1)=20=2\times 10 . So we have a pair ( q , r ) = ( 3 , 11 ) (q,r)=(3,11) of primes. Thus the answer is 3 2 + 1 1 2 + 1 9 2 = 491 3^2+11^2+19^2=\fbox{491} .

Moderator note:

This solution is not clear. Why must we have 20 = 2 × 10 20=2\times 10 instead of 20 = 4 × 5 20=4 \times 5 ?

20 = 2 x 10 isn't make sense. In fact, we must solve the integer equation and chose a prime pairs. So I think other answer like Paola Ramírez or Alexandre Miquilino is more percise

Anh Vũ - 6 years, 4 months ago

Why p=19??????'

Isaac Jiménez - 7 years ago

Log in to reply

Rewrite the given expression: p q r 19 = p + q + r \dfrac{pqr}{19}=p+q+r So 19 p q r 19\mid pqr . Since 19 19 is a prime, by Euclid's lemma, 19 19 divides one of p , q , r p,q,r . Noting the symmetry we WLOG assume 19 p 19\mid p . Since p p is a prime, p = 19 p=19 .

Jubayer Nirjhor - 7 years ago

Log in to reply

what is WLOG?

U Z - 6 years, 4 months ago

Log in to reply

@U Z WLOG means"Without Loss Of Generality".

mudit bansal - 6 years, 4 months ago

because the expression is completely symmetric. No matter which number in three given numbers is equal 19

Anh Vũ - 6 years, 4 months ago

Check again, I think (q-1).(r-1)=18.

Hafizh Ahsan Permana - 6 years, 11 months ago

Log in to reply

No, the solution writer is correct.

We have that q r q r = 19 qr-q-r=19 . Adding one to both sides, we get q r q r + 1 = 20 qr-q-r+1=20 . Factorizing, we conclude ( q 1 ) ( r 1 ) = 20 (q-1)(r-1)=20 .

Daniel Liu - 6 years, 11 months ago

The answer can also be 19 ,21 ,and 2 hence 806

Ritesh Yadav - 6 years, 11 months ago

Log in to reply

@Ritesh Yadav observe that p,q,&r are primes ....... as you state... 21 cannot be prime!

Abhinav Raichur - 6 years, 11 months ago

p q r = 19 ( p + q + r ) pqr = 19(p + q + r)

Either one of p p , q q or r r is equal to 19 19 , given all of them are prime numbers.

WLOG, let's assume p = 19 p = 19 . Then:

q r = 19 + q + r qr = 19 + q + r

q r q = 19 + r qr - q = 19 + r

q = 19 + r r 1 = 1 + 20 r 1 q = \frac {19 + r}{r - 1} = 1 + \frac {20}{r - 1}

Let's test a few values of r r to determine q q .

r = 2 q = 21 r = 2 \rightarrow q = 21 ; untrue because q q is not prime.

r = 3 q = 11 r = 3 \rightarrow q = 11 ; valid solution; both r r and q q are prime.

r = 5 q = 6 r = 5 \rightarrow q = 6 ; untrue because q q is not prime.

r = 7 q = 13 3 r = 7 \rightarrow q = \frac{13}{3} ; untrue because q q is not prime, let alone an integer.

r = 11 q = 3 r = 11 \rightarrow q = 3 ; valid solution, symmetric to the previous one we found with r = 3 r = 3 .

r = 13 q = 8 3 r = 13 \rightarrow q = \frac{8}{3} ; untrue because q q is not prime, let alone an integer.

r = 17 q = 8 3 r = 17 \rightarrow q = \frac{8}{3} ; untrue because q q is not prime, let alone an integer.

r = 19 q = 8 3 r = 19 \rightarrow q = \frac{8}{3} ; untrue because q q is not prime, let alone an integer.

For any r r greater than 21 21 we don't even bother looking for a solution because then q q would not be an integer.

Thus, any solution of the equation given the conditions is a permutation of ( 3 , 11 , 19 ) (3, 11, 19) . Thus, the sum asked equals: 3 2 + 1 1 2 + 1 9 2 = 9 + 121 + 361 = 491 3^{2} + 11^{2} + 19^{2} = 9 + 121 + 361 = 491 .

Nice Explanation...!!

James Kumar - 6 years, 4 months ago

This is exactly how I did it!

Steven Santos - 5 years, 2 months ago
Sswag SSwagf
Sep 23, 2016

Simple sol:

p q r = 19 ( p + q + r ) pqr=19(p+q+r)

p=19

q r = 19 + q + r = qr=19 + q + r=

( q 1 ) ( r 1 ) = 20 (q-1)(r-1)=20

q,r are \le 19

both can only be:

{3,5,7,11,13,17,19}

so

( q 1 ) (q-1) would be only {2,4,6,10,12,16,18}

if q=3 then r=11 both are primes doing this with the other possible results we end up with only {19,11,3}

so result would be 1 9 2 + 1 1 2 + 3 2 = 491 19^{2}+11^{2}+3^{2}=491

Gelvlad Gelvlad
Sep 9, 2015

p should be 19, without loss of generality. Let q=2 m+1 and r=2 n+1 (Neither q nor r can be equal to 2 - the only even prime number, that can be easily checked). Putting the above variables into the given equation results in m*n=5. Considering m and n are positive whole numbers, without the loss of generality we end up with m=1 and n=5, which gives us q=3 and r=11.

Gautam Sharma
Jul 21, 2014

since RHS is amultiple of 19 so lhs should also contain 19 and p,q,r are primes then one of p,q or r should be 19 as 19 is also a prime. Let r be 19 so we are left with (q,p) which can be either (11,3) or (3,11) so p=3 or 11,q=11 or 3 and r=19

So sum of squares =491

Moderator note:

This solution is not entirely clear. You didn't explain how ( 11 , 3 ) (11,3) and ( 3 , 11 ) (3,11) are solutions.

David Krüger
Feb 2, 2016

Used Octave (MatLab) to brute force my way into an answer:

 1
 2
 3
 4
 5
 6
 7
 8
 9
10
11
12
13
14
15
16
17
18
19
20
21
22
23
24
25
26
27
28
29
30
31
32
33
34
35
36
37
38
39
40
41
a = primes(500);

for i = 1:75

    for j = 1:75

        for k = 1:75

            p = a(i);
            q = a(j);
            r = a(k);

            t1 = p*q*r;
            t2 = 19*(p+q+r);

            if t1==t2
                p
                q
                r

                flag=1;

                break
            end

            if flag==1
                break
            end
        end

        if flag==1
            break
        end
    end

    if flag==1
        break
    end
end

(p^2)+(q^2)+(r^2)

1
2
3
4
5
6
7
8
9
OUTPUT:

p =  3

q =  11

r =  19

ans =  491

Would be rather simple to make on TI-basic, too. In this case, i, j, and k can be extended to 95 (not just the 75 they are at now) since there are 95 primes from 1 to 500. I figured that it was already overkill, but with a little patience this could have been used to find much higher primes.

Govind Choudhary
Aug 9, 2018

Just wanted to mention that since there is only one value of p^2 + q^2 + r^2 , we only need to consider one successful case, which, in my case, was (p, q, r) = (19, 3, 11)

Aswin T.S.
Feb 3, 2016

p(qr) =19(p+q+r)

consider p = 19

then qr = p+q+r

since p,q,r are prime nos.

it is possible iff

when q=11,3

      r =3,11

p=19

q=3

r=11

ans =361 +121 + 9

   =491

0 pending reports

×

Problem Loading...

Note Loading...

Set Loading...